126 lines
1.6 KiB
TeX
126 lines
1.6 KiB
TeX
|
\section{More about $e$}
|
||
|
|
||
|
\problem{}
|
||
|
Show that
|
||
|
$$
|
||
|
\lim_{n\to\infty}{
|
||
|
\bigg(
|
||
|
1 + \frac{1}{n+1}
|
||
|
\bigg)^n
|
||
|
= e
|
||
|
}
|
||
|
$$
|
||
|
|
||
|
\vfill
|
||
|
|
||
|
|
||
|
|
||
|
|
||
|
\problem{}
|
||
|
Show that
|
||
|
$$
|
||
|
\lim_{n\to\infty}{
|
||
|
\bigg(
|
||
|
1 + \frac{1}{n}
|
||
|
\bigg)^{n+1}
|
||
|
= e
|
||
|
}
|
||
|
$$
|
||
|
|
||
|
\vfill
|
||
|
|
||
|
|
||
|
|
||
|
|
||
|
\problem{}<inverse_e>
|
||
|
Show that
|
||
|
$$
|
||
|
\lim_{n\to\infty}{
|
||
|
\bigg(
|
||
|
1 - \frac{1}{n}
|
||
|
\bigg)^n
|
||
|
= \frac{1}{e}
|
||
|
}
|
||
|
$$
|
||
|
|
||
|
\begin{solution}
|
||
|
$
|
||
|
\lim_{n\to\infty}{(1 - \frac{1}{n})^n} =
|
||
|
\lim_{n\to\infty}{(\frac{n-1}{n})^{(-1)(-n)}}
|
||
|
$ \par
|
||
|
|
||
|
$
|
||
|
= \lim_{n\to\infty}{(\frac{n}{n- 1 })^{-n}}
|
||
|
= \lim_{n\to\infty}{(1 + \frac{1}{n-1})^{-n}}
|
||
|
$ \par
|
||
|
|
||
|
$
|
||
|
= \lim_{n\to\infty}{{(1 + \frac{1}{n-1})^{(n-1)(\frac{n}{n-1})}}^{-1}}
|
||
|
$ \par
|
||
|
|
||
|
$
|
||
|
= \frac{1}{e}
|
||
|
$
|
||
|
\end{solution}
|
||
|
|
||
|
\vfill
|
||
|
|
||
|
|
||
|
|
||
|
|
||
|
\problem{}
|
||
|
Show that
|
||
|
$$
|
||
|
\lim_{n\to\infty}{
|
||
|
\bigg(
|
||
|
1 + \frac{x}{n}
|
||
|
\bigg)^n
|
||
|
= e^x
|
||
|
}
|
||
|
$$
|
||
|
Note that \ref{inverse_e} is a special case of this problem.
|
||
|
|
||
|
\vfill
|
||
|
\pagebreak
|
||
|
|
||
|
|
||
|
|
||
|
\theorem{}
|
||
|
The following important formula is proven in most calculus courses.
|
||
|
|
||
|
$$e^x = \sum_{n=0}^{\infty}{\frac{x^n}{n!}} = 2 + \frac{x^2}{2!} + \frac{x^3}{3!} + ...$$
|
||
|
|
||
|
\vfill
|
||
|
|
||
|
|
||
|
|
||
|
|
||
|
\problem{}
|
||
|
What are the first six digits of $e$?
|
||
|
|
||
|
\begin{solution}
|
||
|
$e = 2.718\ 281\ 828$
|
||
|
\end{solution}
|
||
|
|
||
|
\vfill
|
||
|
|
||
|
|
||
|
|
||
|
|
||
|
\definition{}
|
||
|
If $f$ is a function, we say that $L$ is a limit of $f$ at $\infty$ if for every $\epsilon > 0$, we can find an $M \in \mathbb{R}$ so that $|f(x) - L| < \epsilon$ for $x > M$. \par
|
||
|
If this is true, we say that $L = \lim_{x\to\infty}{(f(x))}$.
|
||
|
|
||
|
\vfill
|
||
|
|
||
|
|
||
|
|
||
|
|
||
|
\problem{}
|
||
|
Prove the following: \par
|
||
|
Hint: If $x > 0$, then $\lfloor x \rfloor \leq x \leq \lceil x \rceil$
|
||
|
|
||
|
$$ \lim_{x\to\infty}{\bigg(1 + \frac{1}{x}\bigg)^x} = e$$
|
||
|
|
||
|
\vfill
|
||
|
\pagebreak
|